An equilateral triangle with unknown side length

Geometry Level 3

A B C \triangle ABC is an equilateral triangle with side length s s . Point P P is in its interior such that P B = 13 , P C = 14 , P A = 15 \overline{PB} = 13 , \overline{PC} = 14 , \overline{PA} = 15 . If the square of the side length can be expressed as s 2 = a + b c s^2 = a + b \sqrt{c} , where a , b , c a, b, c are positive integers and c c is square-free, find a + b + c a + b + c .

Inspiration: A problem posted by Presh Talwalkar on his channel: Mind Your Decisions

464 463 465 466

This section requires Javascript.
You are seeing this because something didn't load right. We suggest you, (a) try refreshing the page, (b) enabling javascript if it is disabled on your browser and, finally, (c) loading the non-javascript version of this page . We're sorry about the hassle.

3 solutions

Fletcher Mattox
Nov 14, 2020

I haven't looked at the inspiration video yet, but an obvious (to me) solution is via coordinate geometry.

Let B = ( 0 , 0 ) , C = ( s , 0 ) , A = ( s / 2 , s 3 / 2 ) B = (0,0),\enspace C = (s,0),\enspace A= (s/2, s\sqrt{3}/2)

Then P P must lie at the intersection of the circles centered at the vertices:

B: x 2 + y 2 = 1 3 2 \quad x^2 + y^2 = 13^2

C: ( x s ) 2 + y 2 = 1 4 2 \quad(x-s)^2 + y^2 = 14^2

A: ( x s / 2 ) 2 + ( y 3 s / 2 ) 2 = 1 5 2 \quad(x-s/2)^2 + (y - \sqrt{3}s/2)^2 = 15^2

Three equations, three unknowns. solve simultaneously for s s :

s = 295 + 168 3 , s = \sqrt{295 + 168\sqrt{3}},\quad so 295 + 168 + 3 = 466 295+168+3=\boxed{466}

Now let's see how brutish this solution is compared to the inspiration video ...

Chew-Seong Cheong
Nov 15, 2020

Similar solution with @Fletcher Mattox 's but easier computation.

Let the centroid of equilateral A B C \triangle ABC be the origin O ( 0 , 0 ) O(0,0) . Then A ( 0 , s 3 ) A \left(0, \frac s{\sqrt 3} \right) , B ( s 2 , s 2 3 ) B\left(-\frac s2, - \frac s{2\sqrt 3} \right) , and C ( s 2 , s 2 3 ) C\left(\frac s2, - \frac s{2\sqrt 3} \right) . Let P ( x , y ) P(x,y) . Then

{ x 2 + ( y s 3 ) 2 = 1 5 2 . . . ( 1 ) ( x + s 2 ) 2 + ( y + s 2 3 ) 2 = 1 3 2 . . . ( 2 ) ( x s 2 ) 2 + ( y + s 2 3 ) 2 = 1 4 2 . . . ( 3 ) \begin{cases} \begin{aligned} x^2 + \left(y-\frac s{\sqrt 3}\right)^2 & = 15^2 & ...(1) \\ \left(x + \frac s2 \right)^2 + \left(y+\frac s{2\sqrt 3}\right)^2 & = 13^2 & ...(2) \\ \left(x - \frac s2 \right)^2 + \left(y+\frac s{2\sqrt 3}\right)^2 & = 14^2 & ...(3) \end{aligned} \end{cases}

{ ( 2 ) ( 3 ) : 2 s x = 27 x = 27 2 s 2 × ( 1 ) ( 2 ) ( 3 ) : 2 3 s y = 85 y = 85 2 3 s \begin{cases} (2)-(3): & - 2sx = 27 & \implies x = - \dfrac {27}{2s} \\ 2\times (1) - (2)-(3): & - 2\sqrt 3 sy = 85 & \implies y = - \dfrac {85}{2\sqrt 3s}\end{cases}

From ( 1 ) (1) :

2 7 2 4 s 2 + ( 85 2 3 s s 3 ) 2 = 225 2353 3 s 2 85 3 + s 2 3 = 225 2353 s 2 590 + s 2 = 0 s 4 590 s 2 + 2353 = 0 \begin{aligned} \frac {27^2}{4s^2} + \left(-\frac {85}{2\sqrt 3s} - \frac s{\sqrt 3} \right)^2 & = 225 \\ \frac {2353}{3s^2} - \frac {85}3 + \frac {s^2}3 & = 225 \\ \frac {2353}{s^2} - 590 + s^2 & = 0 \\ s^4 - 590s^2 + 2353 & = 0 \end{aligned}

s 2 = 295 ± 168 3 Since 295 168 3 4.015 is too small. = 295 + 168 3 \begin{aligned} \implies s^2 & = 295 \pm 168\sqrt 3 & \small \blue{\text{Since }295 - 168\sqrt 3 \approx 4.015 \text{ is too small.}} \\ & = 295 + 168\sqrt 3 \end{aligned}

Therefore a + b + c = 295 + 168 + 3 = 466 a+b+c = 295 + 168 + 3 = \boxed{466} .

David Vreken
Nov 14, 2020

As hinted in the video, rotate the entire diagram 60 ° 60° clockwise:

Since B C P = A C P \angle BCP = \angle ACP' , and A C B = 60 ° \angle ACB = 60° from equilateral A B C \triangle ABC , that means P C P = 60 ° \angle PCP' = 60° . Also, since C P = C P = 14 CP = CP' = 14 , C P P \triangle CPP' is an isosceles triangle, and since its vertex angle is P C P = 60 ° \angle PCP' = 60° , it is also an equilateral triangle, which means P P = C P = C P = 14 PP' = CP = CP' = 14 .

Let θ = A P P \theta = \angle APP' . By the law of cosines on A P P \triangle APP' , cos θ = P P 2 + A P 2 A P 2 2 P P A P = 1 4 2 + 1 5 2 1 3 2 2 14 15 = 3 5 \cos \theta = \cfrac{PP'^2 + AP^2 - AP'^2}{2 \cdot PP' \cdot AP} = \cfrac{14^2 + 15^2 - 13^2}{2 \cdot 14 \cdot 15} = \cfrac{3}{5} , which means sin θ = 1 cos 2 θ = 4 5 \sin \theta = \sqrt{1 - \cos^2 \theta} = \cfrac{4}{5} .

That means cos A P C = cos ( θ + 60 ° ) = cos θ cos 60 ° sin θ sin 60 ° = 1 10 ( 3 4 3 ) \cos \angle APC = \cos (\theta + 60°) = \cos \theta \cos 60° - \sin \theta \sin 60° = \frac{1}{10}(3 - 4\sqrt{3}) .

Finally, by the law of cosines on A P C \triangle APC , s 2 = C P 2 + A P 2 2 C P A P cos A P C = 1 4 2 + 1 5 2 2 14 15 1 10 ( 3 4 3 ) = 295 + 168 3 s^2 = CP^2 + AP^2 - 2 \cdot CP \cdot AP \cdot \cos \angle APC = 14^2 + 15^2 - 2 \cdot 14 \cdot 15 \cdot \frac{1}{10}(3 - 4\sqrt{3}) = 295 + 168\sqrt{3} .

Therefore, a = 295 a = 295 , b = 168 b = 168 , c = 3 c = 3 , and a + b + c = 466 a + b + c = \boxed{466} .

Use Herons Formula in three of triangle

Devansh Agrawal - 4 months, 2 weeks ago

0 pending reports

×

Problem Loading...

Note Loading...

Set Loading...